All Discussions
HitBox
Word
Docs
Hits
pt10
55
56
s1
1.1K
1.4K
q18
217
231
Related Threads
PT10.S1.Q18 - people cannot be morally responsible
Can anyone help explain how I should translate the stimulus? <br /> I understand why E is the correct answer, but I would love to solidify why this …
PT10.S1.Q18 - people cannot be morally responsible
Can someone help me with this one?<br /> <br /> I don't understand why B is not right. The stimulus I thought was saying that if it's something you…
PT10.S1.Q18 - people cannot be morally responsible
I'm having trouble negating and translating the first sentence of this question. How am I supposed to negate a sentence with two group four indicator…
PT10.S1.Q18 - people cannot be morally responsible
Hey could someone break this down for me? Thanks
PT10.S1.Q18 - people cannot be morally responsible
Hi! <br /> <br /> I am going through MBT and MSS questions because they seem to be my biggest weakness. I came across this question while drilling …
PT10.S1.Q18 - People cannot be morally responsible
Can someone please help me with this question. I've naarrowed it down to A and E. I understand why E is the correct answer but cant figure out why A …
PT10.S1.Q18 - People cannot be morally responsible for
can someone please explain to me how the answer for this question is (e) and not (b)? I've been pondering about it for minutes and I still can't unde…
PT10.S1.Q18 - People cannot be morally responsible for things
I been drilling the question types. Right now I am at MSS/MBT. However, I came across an error that was reoccuring with me in logic. What do you do w…
PT10.S1.Q18 - People cannot be morally responsible
I got this wrong initially by marking down A but the correct answer is E. This stimulus tells us a few things <br /> - people cannot be morally resp…
PT54.S1.Q18 - significance in the history of the cakewalk
How about this question.<br /> <br /> http://7sage.com/lsat_explanations/lsat-54-section-1-passage-3-questions/<br /> <br /> I still confused abo…
Related Searches
pt10 s1 q8
pt10 s1
pt10 s1 q10
pt10 s1 q24
pt75 s1 q18
pt53 s1 q18
pt55 s1 q18
pt5 s1 q18
pt7 s1 q18
pt10 s1 q25
Title Search
@title beer
Word 'beer' in title
User Search
@user admin
Only posts by Admin
Text Search
@body beer
Word 'beer' in the text
Phrase Search
"Hello World"
Exact phrase match
Exclude Terms
car -red
Search cars that are not red
"Or" Searches
honda | bmw
'honda' or 'bmw' results
Multi-Field
@(title,body) hello
Both contain 'hello'
WildCard
hond* civ*
Match all with an astrix
Combination
@title bmw @user admin
See more
supported syntax examples
Search Again :: Adv Search
1000 results in 0.001s
<
1
…
28
29
30
31
32
33
34
>
Relative Support: An Observation/Potential New Trend
Props to the BR Group on this, I really kinda just stole this observation from those guys! So there are two questions in PT 78 where it really fee…
What were you guys wavering between in PT78
S1
Q6? I think the question that was the most WTF for me was S3 Q23.
lol but I'm nervous now bc I don't know if I'm seeing the overall trend - WHAT ELSE AM I MISSING?!
cacrv567
August 2016
General
PT63.
S1
.Q14 -- You know those questions that make you rethink everything?
This question came up on the last BR group call this Wednesday. I'm really curious what you all think about this question! I'd also love to know what…
@nessa.k13.0 said:
I wrote out a page --a love letter so to speak---for this question in an attempt to give myself the best chance of never making the same mistakes that I did on this question again.
This really is a great ...
Cant Get Right
September 2016
Logical Reasoning
For those with accommodated testing for the december lsat....
i was wondering if you guys know how the testing will differ from normal testing. i received an email yesterday from lsac with the following: "We …
... the time you're done
S1
.
inactive
November 2016
General
Establishing a Correlation
I would appreciate if somebody could clarify this one for me.. https://7sage.com/lsat_explanations/lsat-72-section-2-question-14/ Here, JY equate…
... the most automatically" in PT47
S1
Q26 does not establish a ...
SeattlesBest
November 2016
Logical Reasoning
Official December LSAT Discussion Thread
Hey guys! Here's the official December LSAT Discussion Thread. Please keep all discussions of the December 2016 LSAT here! Here's some ground rule…
@sd2813 Yeah lots of disagree q's... I found that too.
I had LR-RC-LR-LG-LR.
Found the first two LR's (
s1
/s3) okay, but after the LG section, I lost my cool for s5 LR.
mcmlaw36
December 2016
December 2016 LSAT
Spoiler Alert: Westworld S01E10 & the LSAT
So, Westworld is a really good show. And it's kinda crazy too. <b>Spoilers starting now:</b> In the season finale of Westworld, the hosts (robots)…
... of "evil" throughout much of
S1
. Her actions in the last ...
SherryS1
December 2016
General
PT07.S3.
Q18
- crop rotation increase yields
Hey All, You can never have too much practice with RC, right? Which is why you should definitely go check out this passage and then help a girl ou…
@bswise2 said:
The problem can be cured by crop rotation, denying the pathogens a suitable host for a period of time."
Hey,
So the reason answer choice "E" works is because if you are rotating crops ...
Sami
January 2017
Reading Comprehension
PT12.
S1
.Q05 - oscar: I have been accused of plagiarizing
So, I've done this question many many times. And I've never felt great about it. Can some run it down for me? Specifically, answer choice A. Here'…
@"The 180 Bro_OVO" said:
I believe A is incorrect because "Writer has right" should not be in the sufficient condition.
Rather, to be correct, A should look like this:
Author granted writer the right ---> Writer has ...
Sami
January 2017
Logical Reasoning
PT20.S4.Q25 - marianne is a professional chess player
Despite reviewing JY's explanation (https://7sage.com/lsat_explanations/lsat-20-section-4-question-25/), I don't understand why answer choice (C) is …
... choice (C) different from PT29 -
S1
- Q16 [https://7sage.com/lsat_explanations ...
Sami
January 2017
Logical Reasoning
Flaw confusion, please help!!!
Hi guys, I have some confusion going on here between the difference of the following question: https://7sage.com/lsat_explanations/lsat-25-se…
... /> The stimulus in on PT27,
S1
, Q23 says:
E ...
The 180 Bro_OVO
February 2017
Logical Reasoning
<
1
…
28
29
30
31
32
33
34
>